Practice questions exam 1 Flashcards

1
Q

A 6 month-old boy is determined to have a systemic malignancy
originating from precursor cells of the nervous system. The
pathology report would state?

A. Adenocarcinoma
B. Neuroblastoma
C. Leukemia
D. Metastatic sarcoma

A

B

How well did you know this?
1
Not at all
2
3
4
5
Perfectly
2
Q

A 48-year-old woman has a routine physical
examination. A 4 cm diameter non-tender mass is
palpated in her right breast. The mass appears fixed to
the chest wall. Another 2 cm non-tender mass is
palpable in the left axillary node. A chest radiograph
reveals multiple 0.5 to 2 cm nodules in both lungs.
Which of the following TNM classifications best
indicates the stage of her disease?
A. T1 N1 M0
B. T1 N0 M1
C. T2 N1 M0
D. T3 N0 M0
E. T4 N1 M1

A

E

How well did you know this?
1
Not at all
2
3
4
5
Perfectly
3
Q

Which of the following is potentially true of a tumor suppressor gene?

A. Allows unrestricted cell growth and proliferation.
B. Promotes different phases of the cell cycle.
C. Produces proteins that block the activity of cyclins.
D. Is often overexpressed in cancer

A

C

How well did you know this?
1
Not at all
2
3
4
5
Perfectly
4
Q

Which phase of the cell cycle do you think is targeted by
Palbociclib (the CDK4/6 inhibitor)?
A. G1
B. S
C. G2
D. M

A

A

How well did you know this?
1
Not at all
2
3
4
5
Perfectly
5
Q

A chemotherapy that interferes with DNA synthesis is

A S-phase specific
B G1-phase specific
C cell cycle non-specific
D M-phase specific

A

A

How well did you know this?
1
Not at all
2
3
4
5
Perfectly
6
Q

What is the most common reason for resistance to multiple
chemotherapies at once?
A. Decreased activation of prodrugs
B. Drug transport out of cells
C. Cell cycle changes
D. Mutations in drug targets

A

B

How well did you know this?
1
Not at all
2
3
4
5
Perfectly
7
Q

Which hormone is produced in the pituitary gland?
A. GnRH
B. LH
C. estrogen
D. progesterone

A

b

How well did you know this?
1
Not at all
2
3
4
5
Perfectly
8
Q

Estrogen receptor primarily binds estrogen where in the cell?
A. On the plasma membrane
B. In the mitochondria
C. In the cytoplasm
D. In the nucleus

A

C

How well did you know this?
1
Not at all
2
3
4
5
Perfectly
9
Q

What enzyme converts androstenedione to estrone?
A. CYP19
B. 5alpha-reductase
C. 17, 20 lyase
D. P450scc

A

A

How well did you know this?
1
Not at all
2
3
4
5
Perfectly
10
Q

Which compound directly inhibits the activity of the estrogen
receptor throughout the body?
A. Letrozole
B. Exemestane
C. Tamoxifen
D. Fulvestrant

A

D

How well did you know this?
1
Not at all
2
3
4
5
Perfectly
11
Q

Which compound is referred to as a SERM?
A. Letrozole
B. Exemestane
C. Tamoxifen
D. Fulvestrant

A

C

How well did you know this?
1
Not at all
2
3
4
5
Perfectly
12
Q

What is unique about the action of the Tamoxifen as compared to
Fluvestrant?
a) It leads to ER degradation
b) It holds ER out of the nucleus
c) It ejects ER from the cell
d) It activates ER in bone

A

D

How well did you know this?
1
Not at all
2
3
4
5
Perfectly
13
Q

Which of the following is not a hormone responsive cancer type?
a) Breast cancer
b) Ovarian cancer
c) Prostate Cancer
d) Endometrial cancer

A

B

How well did you know this?
1
Not at all
2
3
4
5
Perfectly
14
Q

Which of the following is only used in the postmenopausal setting?
a) Letrozole
b) Tamoxifen
c) Leuprolide
d) Raloxifene

A

A

How well did you know this?
1
Not at all
2
3
4
5
Perfectly
15
Q

Which compound acts directly on AR?
a) Leuprolide
b) Abiraterone
c) Degarelix
d) Enzalutamide

A

D

How well did you know this?
1
Not at all
2
3
4
5
Perfectly
16
Q

Which amino acid is not a target of phosphorylation?
A. Tyrosine
B. Serine
C. Threonine
D. Alanine

A

D

How well did you know this?
1
Not at all
2
3
4
5
Perfectly
17
Q

What is the source of the phosphate the gets transferred onto a
substrate by a kinase?
A. SAM
B. DNA
C. RNA
D. ATP

A

D

How well did you know this?
1
Not at all
2
3
4
5
Perfectly
18
Q

Which compounds inhibit EGFR?
A. Gefitinib
B. Osmertinib
C. Afatinib
D. Lapatinib
E. All of the above

A

E

How well did you know this?
1
Not at all
2
3
4
5
Perfectly
19
Q

Which of the following drugs targets Her2?
a) Sirolimus
b) Alectinib
c) Vemurafinib
d) Tucatinib

A

D

How well did you know this?
1
Not at all
2
3
4
5
Perfectly
20
Q

What mutation in EGFR confers resistance to 1st and 2nd generation
EGFR inhibitors?
A. L858R
B. Exon 19 deletion
C. Exon 14 deletion
D. T790M

A

D

How well did you know this?
1
Not at all
2
3
4
5
Perfectly
21
Q

Which of the following drugs targets a kinase that is produced by
formation of the Philadelphia chromosome?
a) Alectinib
b) Gefitinib
c) Imatinib
d) binimetinib

A

c

How well did you know this?
1
Not at all
2
3
4
5
Perfectly
22
Q

Which compound below is not a covalent kinase inhibitor?
A. Gefitinib
B. Osimertinib
C. Afatinib
D. Acalabrutinib

A

A

How well did you know this?
1
Not at all
2
3
4
5
Perfectly
23
Q

Which type of kinase inhibitor can bind in the ATP binding site and
stabilize the inactive confirmation of a kinase?
A. Type I
B. Type II
C. Type III
D. Type IV

A

B

How well did you know this?
1
Not at all
2
3
4
5
Perfectly
24
Q

Which drug is a NUCLEOSIDE analog
A. 5-FU
B. Cytarabine

A

B

How well did you know this?
1
Not at all
2
3
4
5
Perfectly
25
Q

Which drug primarily interferes with purine biosynthesis?
A. 5-Fluorouracil
B. Capecitabine
C. 6-Mercaptopurine
D. Cytosine arabinoside

A

C

How well did you know this?
1
Not at all
2
3
4
5
Perfectly
26
Q

Anti-folates all share the following characteristics except:
A. They inhibit DHFR
B. They cause myelosuppression
C. They induce a G2/M cell cycle block
D. The toxic effects are “rescued” by Leucovorin

A

C

How well did you know this?
1
Not at all
2
3
4
5
Perfectly
27
Q

What rescues a 5-FU overdose?
A. Leucovorin
B. Cytosine arabinoside
C. Thymidine
D. Methotrexate

A

C

How well did you know this?
1
Not at all
2
3
4
5
Perfectly
28
Q

What increases the efficacy of 5-FU?
A. Leucovorin
B. Cytosine arabinoside
C. Thymidine
D. Methotrexate

A

A

How well did you know this?
1
Not at all
2
3
4
5
Perfectly
29
Q

Which antimetabolite should not be given to patients being
treated for gout?
A. Cytosine Arabinoside
B. 6-mercaptopurine
C. Cladribine
D. 5-FU

A

B

How well did you know this?
1
Not at all
2
3
4
5
Perfectly
30
Q

Which anti-metabolite does not directly inhibit DNA/RNA
synthesis?
A. 5-fluorouracil
B. Cytosine Arabinoside
C. Nelarabine
D. Methotrexate

A

D

How well did you know this?
1
Not at all
2
3
4
5
Perfectly
31
Q

Alkylating agents are potent…
A. Reducing agents
B. Electrophiles
C. Nucleophiles
D. Oxidizing agents

A

B

How well did you know this?
1
Not at all
2
3
4
5
Perfectly
32
Q

Which of the following is a prodrug?
1. Mechlorethamine
2. Cyclophosphamide
3. Mitomycin C
4. Chlorambucil

A

2

How well did you know this?
1
Not at all
2
3
4
5
Perfectly
33
Q

To block hemorrhagic cystitis, Mesna is coadministered with
which drug?
A. Methotrexate
B. Cyclophosphamide
C. Cisplatin
D. Mitomycin C

A

B

34
Q

Which of the following is NOT a crosslinker?
A. Cytarabine
B. Cyclophosphamide
C. Carboplatin
D. Chlorambucil

A

A

35
Q

Topoisomerase I inhibitors primarily halt cells in which phase of the
cell cycle?
A. G0/G1
B. S
C. G2
D. M

A

S

36
Q

A patient has a history of heart disease and poor cardiac function.
Which topoisomerase inhibitor should not be prescribed?
A. Doxorubicin
B. Etoposide
C. Irinotecan

A

A

37
Q

Which of the following microtubule inhibitors block the
polymerization of tubulin?
A. Paclitaxel
B. Vincristine
C. Ixabepilone

A

B

38
Q

Which of the following is used to describe a cancer derived from
pigment producing cells of the eye?
a) Teratoma
b) Papilloma
c) Sarcoma
d) Melanoma
e) All of the above.

A

D

39
Q

Which of following is not a hallmark of cancer?
a) Resisting cell death
b) Evading Growth suppressors
c) Enabling replicative immortality
d) Activating invasion and metastasis
e) Increasing in cell size

A

E

40
Q

Molecular pathology is different from traditional pathology
because…?
a) It uses a numerical system to categorize tumor
progression.
b) It uses some kind of genetic testing to indicate therapy or
predict for tumor recurrence.
c) It uses stains that allow pathologists visualize nuclear and
cytoplasmic size and shape.
d) It is not useful when determining a patient’s course of
treatment.

A

B

41
Q

Which of the following terms is associated with substitution of
tissue type for another?
a) Hyperplasia
b) Dysplasia
c) Metaplasia
d) Desmoplasia

A

C

42
Q

Which is of the following is an example of a tumor suppressor?
a) BRCA
b) HER2
c) CDK4/6
d) HPV

A

A

43
Q

Which of the following antibodies targets Her2?
a) Neratinib
b) Cetuximab
c) Lapatinib
d) Pertuzumab

A

D

44
Q

What was the first immunotherapy?
a) Pertuzumab
b) Keytruda
c) Coley’s toxin
d) Trastuzumab

A

C

45
Q

he antibody Fakeumab has just entered the market. What type of the
antibody was it?
a) Fully human
b) Humanized
c) Fully mouse
d) Chimeric

A

A

46
Q

Describe why the combined use pertuzumab and trastuzumab both
defies and complies with general rules of combination therapy?

A

Against: Shared target equals shared toxicities.
For: Different bindings sites on HER2, different mechanisms of HER2
inhibition, enhanced activation of the ADCC.

47
Q

Which antibody is effective against metastatic melanoma and
targets CTLA-4?
A. Ipilimumab
B. Trastuzumab
C. Daratumumab
D. Blinatumomab

A

A

48
Q

Which of the following proteins is used as a cancer biomarker test
to determine eligibility for treatment of NSLCL with pemrolizumab?
A. CD19
B. CTLA4
C. PDL1
D. PD1

A

C

49
Q

Which of the following antigens is a common target for CAR-T
therapy in B cell malignancy?
A) CD4
B) CD19
C) CD28
D) CTLA4

A

B

50
Q

What is the primary mechanism of action of bispecific T cell
engagers in cancer immunotherapy?
A) They inhibit the PD1/PDL1 interaction
B) They bring T cells and cancer cells in close proximity to facilitate
T cell mediated cytotoxicity
C) They stimulate the innate immune system to recognize tumor
cells.
D) They increase neoantigens.

A

B

51
Q

Which of the following drugs acts by binding to HER2?
a) Trastuzumab
b) Pertuzumab
c) Margetuximab
d) All of the above

A

D

52
Q

Which of the following drugs exerts antitumor effects by inhibiting the
interaction between PD1 and PDL1?
a) Cetuximab
b) Ipilimumab
c) Atezolizumab
d) All of the above

A

C

53
Q

What concept of immunity must be overcome for the immune system to
successfully eliminate cancer cells?
a) Antibody-dependent cellular cytotoxicity
b) Central tolerance
c) T-cell activity
d) B-cell activity

A

B

54
Q

What is the function of the cetuximab?
a) To block EGFR ligand binding
b) The same as panitumumab
c) To binding VEGF
d) The same at Pertuzumab
e) A and B

A

E

55
Q

Which of the following is a topoisomerase inhibitor?

Question options:

Irinotecan

Methotrexate

Paclitaxel

Carboplatin

A

Irinotecan

56
Q

Chemotherapy can be selective for cancers cells over normal cells for what reasons?

Some cancers grow faster than normal cells

Some cancers have defective cell cycle checkpoints

Some cancer selectively activate prodrugs of chemotherapy agents

All of the above

A

ALL OF THE ABOVE

57
Q

Which drug targets the kinases directly involved in cell cycle control?

Lapatinib

Olaparib

Dabrafenib

Palbociclib

A

Palbociclib

58
Q

What is the most common DOSE-LIMITING side effect for cancer therapeutics?

Neuropathy

Myelosuppression

Nephrotoxicity

Rash

A

Myelosuppression

59
Q

One of the most common forms of leukemia is Chronic Myelogenous Leukemia (CML). In most cases these leukemic cells contain a chromosomal abnormality that is a “target” for drug therapy that is not seen in non-leukemic white blood cells. A current treatment of choice for this form of cancer is:

Imatinib

Trastuzumab

Bortemzomib

Tamoxifen

A

Imatinib

60
Q

SM is a 52-year-old man with a new diagnosis of metastatic lung adenocarcinoma. Complete pathologic review revealed a translocation of EML4-ALK. Which therapy would be prescribed?

Imatinib

Gefitinib

Alectinib

Quizartinib

A

Alectinib

61
Q

A postmenopausal woman is diagnosed with ER-/HER2+ breast cancer and is treated with standard of care first-line therapy. First line therapy here could consist of which two antibodies?

A bispecific antibody and an antibody that binds the TCR

Trastuzumab and Rituximab

Pertuzumab and Bevacizumab

Pertuzumab and Trastuzumab

A

Pertuzumab and Trastuzumab

62
Q

Which of the following is not a cell type found in solid tumors?

Blood vessels

T-cells

Tumor cells

All are found in solid tumors

A

All are found in solid tumors

63
Q

Which of following agents is appropriate regarding the treatment of women with ER-positive breast cancer?

Abiraterone may be used in pre and postmenopausal women

Tamoxifen may be used in pre and postmenopausal women

Enzalutamide may be used in pre and postmenopausal women

Letrozole may be used in pre and postmenopausal women

A

Tamoxifen may be used in pre and postmenopausal women

64
Q

The use of which drug is based on the concept of synthetic lethality?

Olaparib

Methrotexate

Cisplatin

Irinotecan

A

Olaparib

65
Q

Pituitary secretion of which molecule stimulates aromatase expression in the ovaries?

A. GnRH
B. FSH
C. Progesterone
D. Megestrol

A

B

66
Q

The T-cell receptor binds to a specific peptide presented by an MHC molecule in the context of coactivating signals. Which of the following is not one of the components of the 3rd generation chimeric antigen receptor that can substitute for these co-activating signals?

A. A single chain antibody for CD19
B. Components of the 4-1BB receptor
C. Components of CTLA4
D. Components of CD28

A

C

67
Q

A patient’s pathology report indicates their tumor is an adenoma with a high degree of dysplasia. What does this mean?

A. The tumor is small and no sign of local invasion, tumor cells were found in the lymph nodes and metastases were detected.
B. The tumor is derived from secretory cells and there is a loss of normal tissue architecture.
C. The tumor is pushing into surrounding tissue, tumor cells were from pigment producing cells.
D. The tumor is growing in stalk-like projections and incorporated a high degree of connective tissue.

A

B

68
Q

You have a patient who has just been diagnosed with melanoma. A molecular diagnostic indicates presence of a BRAF V600E mutation. Which of the following drugs would most likely be indicated?
A. Lapatinib in combination with Idelalisib
B. Dabrafenib in combination with binimetinib
C. Pertuzumab in combination with trastuzumab
D. Afatinib in combination with Gefitinib

A

B

69
Q

Which of the following therapies recruits T-cells to kill B cells?

A. trastuzumab
B. Ipilimumab
C. Cetuximab
D. Blinatumomab

A

D

70
Q

Which of the following drugs demonstrates preferential inhibition of HER2 as compared to other members of the ErbB family?

A. Tucatinib
B. Gefitinib
C. Osimertinib
D. Afatinib

A

A

71
Q

Other than CML, what is another indication for Imatinib? Why?

A. GIST-Imatinib also inhibits cKit which is commonly mutated in GIST
B. Small cell lung cancer- Imatinib also inhibits EGFR which is commonly mutated in lung cancer
C. Hormone refractory breast cancer- Imatinib also inhibits HER2 which is commonly amplified in breast cancer
D. Hormone refractory prostate cancer- Imatinib reactivates androgen receptor, resensitizing prostate cancers to hormone therapy.

A

A

72
Q

You have a breast cancer patient whose tumor has tested positive for an EGFR mutation. Which of the following is a covalent inhibitor that could be indicated?

A. Tucatinib
B. Afatinib
C. Binimetinib
D. Acalabrutinib

A

B

73
Q

What enzyme converts Androstenedione to Estrone?

A. 5-reductase
B. PI3 Kinase
C. 17-hydroxylase
D. Aromatase

A

D

74
Q

Vincristine induces which cell cycle checkpoint?

A. G1/G0
B. S
C. G2/M
D. cell cycle non-specific

A

C

75
Q

Which of the following chemotherapies does not directly cause DNA damage?

A. Doxorubicin
B. Paclitaxel
C. Mitomycin C
D. Cisplatin

A

B

76
Q

Identify which of the following chemotherapy drugs is paired with a drug specifically intended to alleviate its toxic side effects.

A. Cytarabine; leucovorin
B. Doxorubicin; Dexrazoxane
C. Vincristine; MESNA
D. 5-FU: chlorambucil

A

B

77
Q

A patient is on standard combination chemotherapy for non-Hodgkin’s lymphoma. In 5 years he develops AML. Which of the four chemotherapies in this combination chemotherapy is most likely the cause of this secondary malignancy?

A. Cyclophosphamide
B. Methotrexate
C. Vincristine
D. Prednisolone

A

A

78
Q

When combined with leucovorin, the efficacy of this drug against cancer is increased (Consider the mechanism of action).

A. Vincristine
B. Bleomycin
C. Doxorubicin
D. 5-Fluorouracil

A

D

79
Q

The most common dose-limiting side effect of chemotherapy drugs are:

A. gingival hyperplasia
B. Peripheral neuropathy
C. myelosuppression
D. Cardiac toxicity

A

C

80
Q

MESNA is used to block the hemorrhagic cystitis caused by which common chemotherapy drug?
A. Doxorubicin
B. Cisplatin
C. Cyclophosphamide
D. 5-FU

A

C

81
Q

Based on the principles of combination chemotherapy, which of the following is best to add to a cocktail of Cyclophosphamide and Doxorubicin?

A. Etoposide
B. Methotrexate
C. Chlorambucil
D. Mitoxantrone

A

B